Find a1 for the arithmetic sequence's 21st term is 400 is 400 and it's common difference is 5​

Answers

Answer 1

Answer:

8,395

Step-by-step explanation:

21 x 400 = 8,400

is = x

8, 400 - 5 = 8,395

difference = -

Brainlist Pls!


Related Questions

. In a volleyball game, Alexis scored 4 points more than twice the
number of points Jessica scored. Jessica scored 3 points. How many
points did Alexis score?
F. 1 point G. 7 points H. 10 points I. 12 points

Answers

Answer: 10

Step-by-step explanation:

Alexis Scored 4 more than twice the number of points Jessica scored.

Jessica scored 3

twice the number of 3 would be 3 x 2 which equals six

4 more than twice the number which is 6 would be 10, 4+6=10

An insurance policy sells for $600. Based on past data, an average of 1 in 50 policyholders will file a $5,000 claim, and average of 1 in 100 policyholders will file a $10,000 claim, and an average of 1 in 200 policyholders will file a $30,000 claim. What is the expected value per policy sold?

Answers

Answer:

$250

Step-by-step explanation:

Calculation to determine the expected value per policy sold

Expected value​ per policy sold =$600-(1/50)*$5,000-(1/100)*$10,000-(1/200)*$30,000

Expected value​ per policy sold =$600-$100-$100-$150

Expected value​ per policy sold =$250

Therefore the expected value per policy sold will be $250

A recipe uses 6 tablespoons of butter for every 8 oz of cheese. the rate is __ tablespoons for every 1 oz. the raze is __ oz for every 1 tablespoon. ​

Answers

1/4 or .75

6 divided by 4 equal 1/4 or .75

find the missing side x​

Answers

Answer:

[tex]\sqrt{968}[/tex]

Step-by-step explanation:

Since this is a right triangle, we are able to use pythagorean theorem, a^2+b^2=c^2. In this case x would be the "c", so 22^2+22^2=x^2. Isolate the variable and solve for x. 484+484=x^2

968=x^2

[tex]\sqrt{968\\}[/tex]=x

help I will give brainiest if you can atleast do three​

Answers

1.) A=pi(r)^2

2.) V=Bh

3.) 3.1

4.) 20

5.)36

6.) 10

7.)18

Answer:

Step-by-step explanation:

1.) formula of circle =pi times r^2

2.)volume of cylinder =pi times r^2 times h

3.)value of pi rounded = 3.14

4.) diameter of can A =2r=2(10) = 20

diameter of Can A is 20

5.)diameter of can B =2r =2(18) =36

diameter of Can B is 36

6.)radius = 10

7.)radius =18

I did all of them.

A soccer field is 100 meters long. What could be its length in yards? A. 33.3 B. 91 C. 100 D. 109​

Answers

Answer:

D. 109

Step-by-step explanation:

100 meters into yards is 109.361

the ratio is 1.094 so 109

How many solutions does this equation have? 8 + 10z = 3 + 9z
-no solution
-one solution
-infinitely many solutions

Answers

one solution
because z ONLY = -5

A Store owners offers a discount of 20% off the regular price of all jackets. Jessica has a coupon that gives her an additional 5% off the discount price. The original price of jacket Jessica buys is $84. What is the price of the jacket after the discount and Jessica coupon?

Answers

Answer:

$63

Step-by-step explanation:

The store is 20% off, Jessica has a coupon that is 5% off add that together and it's 25% off. $84 - 25% = $63

let $p$ and $q$ be the two distinct solutions to the equation$$\frac{4x-12}{x^2 2x-15}=x 2.$$if $p > q$, what is the value of $p - q$?

Answers

let $p$ and $q$ be the two distinct solutions to the equation$$\frac{4x-12}{x^2 2x-15}=x 2.$$if $p > q$. The value of $p - q$ is 4.

To find the value of $p - q$, we first need to solve the given equation and determine the values of $p$ and $q$.

The equation is:

$$\frac{4x-12}{x^2 - 2x - 15} = x^2.$$

Step 1: Factorize the denominator:

The denominator can be factored as $(x - 5)(x + 3)$.

Step 2: Simplify the equation:

$$\frac{4x-12}{(x - 5)(x + 3)} = x^2.$$

Step 3: Multiply both sides of the equation by $(x - 5)(x + 3)$ to eliminate the denominator:

$$(4x - 12) = x^2(x - 5)(x + 3).$$

Step 4: Expand and rearrange the equation:

$$4x - 12 = x^4 - 2x^3 - 15x^2 + 25x.$$

Step 5: Rearrange the equation and combine like terms:

$$x^4 - 2x^3 - 15x^2 + 21x - 12 = 0.$$

Step 6: Factorize the equation:

$$(x - 3)(x + 1)(x - 2)(x + 2) = 0.$$

From this, we get four possible solutions: $x = 3$, $x = -1$, $x = 2$, and $x = -2$.

However, we are interested in the two distinct solutions $p$ and $q$, where $p > q$. Therefore, the values of $p$ and $q$ are $p = 3$ and $q = -1$.

Finally, we can find the value of $p - q$:

$$p - q = 3 - (-1) = 3 + 1 = 4.$$

Hence, the value of $p - q$ is 4.

Learn more about the denominator:

https://brainly.com/question/1217611

#SPJ11

In a large population of adults, the mean IQ is 111 with a standard deviation of 22. Suppose 55 adults are randomly selected for a market research campaign. (Round all answers to 4 decimal places, if needed.)

(a) The distribution of IQ is approximately normal is exactly normal may or may not be normal is certainly skewed.

(b) The distribution of the sample mean IQ is approximately normal exactly normal not normal left-skewed right-skewed with a mean of ? and a standard deviation of .

(c) The probability that the sample mean IQ is less than 107 is .

(d) The probability that the sample mean IQ is greater than 107 is .

(e) The probability that the sample mean IQ is between 107 and 117 is

Answers

(a) The distribution of IQ is approximately normal.

(b) The distribution of the sample mean IQ is approximately normal with a mean of 111 and a standard deviation of 3.0410.

(c) The probability that the sample mean IQ is less than 107 is 0.1056.

(d) The probability that the sample mean IQ is greater than 107 is 0.8944.

(e) The probability that the sample mean IQ is between 107 and 117 is 0.7881.

How to solve the research campaign?

(a) The given information does not indicate any significant departure from normality, and with a large population, the Central Limit Theorem suggests that the distribution of IQ will be approximately normal.

(b) The mean of the sample mean IQ will be equal to the population mean, which is 111.

The standard deviation of the sample mean can be calculated by dividing the population standard deviation (22) by the square root of the sample size (55).

Therefore, the standard deviation of the sample mean IQ is 22 / √(55) = 3.0410.

(c) To calculate this probability, standardize the sample mean IQ value using the formula z = (x - μ) / (σ / √(n)),

where x = value to find the probability for, μ = population mean, σ = population standard deviation, and n = sample size.

In this case, find the probability for x = 107.

By plugging in the values, calculate the z-score and then use a standard normal distribution table or calculator to find the corresponding probability, which is 0.1056.

(d) Similar to part (c), use the same formula to standardize the value of 107 and calculate the z-score.

Then, find the probability of the sample mean IQ being greater than 107 by subtracting the probability found in part (c) from 1, which is 0.8944.

(e) To calculate this probability, find the individual probabilities for both values and then subtract the probability found in part (d) from the probability found in part (c).

The probability of the sample mean IQ being less than 107 is 0.1056, and the probability of the sample mean IQ being greater than 117 is 0.2119 (which can be found by subtracting the probability of being less than 117 from 1).

Therefore, the probability of the sample mean IQ being between 107 and 117 is 0.1056 + (1 - 0.2119) = 0.7881.

Find out more on probability here: https://brainly.com/question/25870256

#SPJ1

Students deliver catalogues and leaflet to houses. One day they have to deliver 384 catalogues and 1890 leaflets. Each student can deliver either 16 catalogues or 90 leaflets in hour. Each student can only work for 1 hour. All students hired are paid £51.30 per day, even if they don't work a full day. If the minimum number of wages are hired, how much will the wage bill be

Answers

Answer:

£2308.5 per day

Step-by-step explanation:

Since in one day they have to deliver 384 catalogues and 1890 leaflets and each student can deliver either 16 catalogues or 90 leaflets in hour, the amount of students required to deliver 384 catalogues in one hour is 384/16 =  24 students.

Also, the number of students required to deliver 1890 leaflets in one hour is 1890/90 = 21 students.

So the total number of students required to make the delivery is thus 24 + 21 = 45 students. This is the minimum number of students required for the delivery.

Since all students hired are paid £51.30 per day, even if they don't work a full day, so the amount of wage paid for this minimum amount of students is thus minimum amount × wage = 45 × £51.30 per day = £2308.5 per day

Answer:

£307.80

Step-by-step explanation:

Wow seems the verified answer is wrong.

Who would've thought?

16*384=24

90*1890=21

21+24=45

45/8=5.62500

5.625 rounds to 6

51.30*6=£307.80

Thats your working out

You have to divide 45 by 8 because there are 8 hours in a day in which they can work.

Then round the number as you cant have a fraction of a person

Which you would then multiply by 51.30

Brainliest would be appreciated <33

Hope it helps!

The table below lists the number of games played in a yearly​ best-of-seven baseball championship​ series, along with the expected proportions for the number of games played with teams of equal abilities. Use a 0.05 significance level to test the claim that the actual numbers of games fit the distribution indicated by the expected proportions.
Games_Played Actual_contests Expected_proportion
4 16 0.125
5 21 0.25
6 21 0.3125
7 38 0.3125
determine the null hypotheses
what is the t statistics
what is the p value
what is the conclusion for the test statistic

Answers

The null hypothesis is that the actual numbers of games fit the distribution indicated by the expected proportions.

The following is the calculation of the t-statistics for the given data.

[tex]T=\frac{(O_i-E_i)} {\sqrt{E_i}} [/tex]where [tex]O_i[/tex] represents the observed frequency, and [tex]E_i[/tex] represents the expected frequency.t statistics for

4: [tex]\frac {(16-25)} {\sqrt {25(0.125)}} [/tex] = -3.2t statistics for

5: [tex]\frac {(21-25)} {\sqrt {25(0.25)}} [/tex] = -1.8t statistics for

6: [tex]\frac {(21-31)} {\sqrt {25(0.3125)}} [/tex] = -3.2t statistics for

7: [tex]\frac {(38-31)} {\sqrt {25(0.3125)}} [/tex] = 3.2

The critical value of t at the 0.05 level of significance is ± 2.132. Since the t-statistics of 3.2 > 2.132, we reject the null hypothesis. So, there is a significant difference between the actual number of games played and the expected number of games played in the baseball championship series at the 0.05 significance level. p-value = P (|t| > 3.2) = 0.002

The conclusion for the test statistic: Since the p-value (0.002) is less than the level of significance (0.05), we reject the null hypothesis and conclude that there is a significant difference between the actual numbers of games played and the distribution indicated by the expected proportions.

To know more about statistics, refer to:

https://brainly.com/question/27342429

#SPJ11

The answers are:
5
10
35
55
please help

Answers

Answer:

35.

Step-by-step explanation:

can i get brainliesttt

jus solved it.

simplify this answer pls

Answers

Answer:

D

Step-by-step explanation:

when it's a power of the power we multiply the powers to get a single value for the power.

(6^(1/4))^4=6^(4*(1/4)) (4*(1/4)=1)

=6^1=6

so the answer is D

Equation in slope intercept form that represents their shown

Answers

Answer:

I think the answer would be Y= -2X+5 .

Hope it helps u ^^♥️

find w such that 2u v − 3w = 0. u = (−6, 0, 0, 2), v = (−3, 5, 1, 0)

Answers

To find the value of w that satisfies the equation 2u v - 3w = 0, where u = (-6, 0, 0, 2) and v = (-3, 5, 1, 0), we can substitute the given values into the equation and solve for w.

Substituting the given values of u and v into the equation 2u v - 3w = 0, we have:

2(-6, 0, 0, 2)(-3, 5, 1, 0) - 3w = 0.

Expanding the scalar multiplication and performing the dot product, we get:

(-12, 0, 0, 4)(-3, 5, 1, 0) - 3w = 0,

(36 + 0 + 0 + 0) - 3w = 0,

36 - 3w = 0.

Simplifying the equation, we have:

36 = 3w,

w = 12.

Therefore, the value of w that satisfies the equation is 12. By substituting w = 12 into the equation 2u v - 3w = 0, we get:

2(-6, 0, 0, 2)(-3, 5, 1, 0) - 3(12) = 0,

(-12, 0, 0, 4)(-3, 5, 1, 0) - 36 = 0,

36 - 36 = 0,

0 = 0.

Hence, the value of w = 12 makes the equation true, satisfying the given condition.

Learn more about  scalar multiplication here:

https://brainly.com/question/28875206

#SPJ11

There’s a picture of my question plz help :)

Answers

Answer:

1,534 inches squared

Step-by-step explanation:

To find surface area we just solve for the area of all the sides and add those together. A rectangular prism (a box like above) has 6 sides. There are...

2 sides each of the following dimensions:

2(13×26)=

2(338)=676

2(13×11)=

2(143)=286

2(26×11)=

2(286)=572

Add the area of all 6 sides...

676+286+572=1,534

Remember it is squared not cubed.

The playground at a park is shaped like a trapezoid the dimensions what is the area of the playground in square feet

Answers

Answer:

[tex]Area = 1560ft^2[/tex]

Step-by-step explanation:

Given

See attachment for playground

Required

Determine the area

The playground is a trapezoid. So;

[tex]Area = \frac{1}{2}(Sum\ parallel\ sides) * Height[/tex]

From the attachment, the parallel sides are: 68ft and 36ft

The height is: 30ft

So, the area is:

[tex]Area = \frac{1}{2}(68ft + 36ft) * 30ft[/tex]

[tex]Area = \frac{1}{2}(104ft) * 30ft[/tex]

[tex]Area = 52ft * 30ft[/tex]

[tex]Area = 1560ft^2[/tex]

An after school music program has 15 out 50 students practicing. Write 15/50 (15 over 50) as a decimal and as a percent.

Decimal -
Percent -​

Answers

15/50 * 2/2 = 30/100
Now you can easily see decimal and percent form

Decimal: 0.30
Percent: 30 percent

Answer:

Percent- 30

decimal-0.3

Step-by-step explanation:

Hope this helps and have a wonderful day!!!

The design for the palladium window shown includes a semicircular shape at the top. The bottom is formed by squares of equal size. A shade for the window will extend 4 inches beyond the perimeter of the window, shown by the dashed line around the window. Each square in the window has an area of 169 in2. Round your answers to the nearest whole number.

Answers

Answer:

[tex](a)\ Area = 3765.32[/tex]

[tex](b)\ Area = 4773[/tex]

Step-by-step explanation:

Given

[tex]A_1 = 169in^2[/tex] --- area of each square

[tex]Shade = 4in[/tex]

See attachment for window

Solving (a): Area of the window

First, we calculate the dimension of each square

Let the length be L;

So:

[tex]L^2 = A_1[/tex]

[tex]L^2 = 169[/tex]

[tex]L = \sqrt{169[/tex]

[tex]L=13[/tex]

The length of two squares make up the radius of the semicircle.

So:

[tex]r = 2 * L[/tex]

[tex]r = 2*13[/tex]

[tex]r = 26[/tex]

The window is made up of a larger square and a semi-circle

Next, calculate the area of the larger square.

16 small squares made up the larger square.

So, the area is:

[tex]A_2 = 16 * 169[/tex]

[tex]A_2 = 2704[/tex]

The area of the semicircle is:

[tex]A_3 = \frac{\pi r^2}{2}[/tex]

[tex]A_3 = \frac{3.14 * 26^2}{2}[/tex]

[tex]A_3 = 1061.32[/tex]

So, the area of the window is:

[tex]Area = A_2 + A_3[/tex]

[tex]Area = 2704 + 1061.32[/tex]

[tex]Area = 3765.32[/tex]

Solving (b): Area of the shade

The shade extends 4 inches beyond the window.

This means that;

The bottom length is now; Initial length + 8

And the height is: Initial height + 4

In (a), the length of each square is calculated as: 13in

4 squares make up the length and the height.

So, the new dimension is:

[tex]Length = 4 * 13 + 8[/tex]

[tex]Length = 60[/tex]

[tex]Height = 4*13 + 4[/tex]

[tex]Height = 56[/tex]

The area is:

[tex]A_1 = 60 * 56 = 3360[/tex]

The radius of the semicircle becomes initial radius + 4

[tex]r = 26 + 4 = 30[/tex]

The area is:

[tex]A_2 = \frac{3.14 * 30^2}{2} = 1413[/tex]

The area of the shade is:

[tex]Area = A_1 + A_2[/tex]

[tex]Area = 3360 + 1413[/tex]

[tex]Area = 4773[/tex]

if i do something to the numerator of a fraction, am i supposed to do the same to the denominator too? and if yes,why?

for example i want to multiply 2/2 over 6/2, is it necessary to multiply 2/2 or can I just multiply 2?​

Answers

Step-by-step explanation:

When performing operations on fractions, it is important to maintain the relationship between the numerator and the denominator. In general, if you do something to the numerator, you should also do the same to the denominator.

In your example, if you want to multiply the fraction 2/2 by 6/2, it is necessary to multiply both the numerator and the denominator by the same value. Here's why:

When you multiply fractions, you multiply the numerators together and the denominators together. So, in this case, the multiplication would be:

(2/2) * (6/2) = (2 * 6) / (2 * 2) = 12/4

If you had only multiplied the numerator (2) by 6, the result would have been:

(2 * 6) / 2 = 12/2

As you can see, these two results are different. The correct result is 12/4, which simplifies to 3/1 or simply 3. If you only multiplied the numerator, you would have obtained 12/2, which simplifies to 6.

So, it's necessary to apply the same operation (in this case, multiplication by 2) to both the numerator and the denominator in order to maintain the value of the fraction.

Which statements are correct? Check all that apply. 5 students study both French and Spanish. 63 students study French. 2 students study neither French nor Spanish. 30 students study French, but not Spanish. 63 students study Spanish.

Answers

Answer:

The correct answers are:

• Option 1,

• Option 3 and

• Option 4

Step-by-step explanation:

From the two-way table, it is true that;

1.) 5 students study both French and Spanish.

2.) 30 students study French but not Spanish.

3.) 2 students study neither French nor Spanish

4.) 35 students study French

5.) 68 students study Spanish

6.) 63 students study Spanish but not French

This makes the correct answers options 1, 3 and 4

P.S: I believe your question is from the attached image

Answer:

A. 5 students study both French and Spanish.

C. 2 students study neither French nor Spanish.

D. 30 students study French, but not Spanish.

:)

Can anyone please help asap I keep getting those links bots

Answers

I don’t understand the question 14=7

please help, tysm for your assistance if you do :)

Answers

Answer:

27/49

plz mark me as brainliest

For a random variable X where X ~ N(p, p(1-p)/k) and 0<=p<=1, find the value of k for which X will estimate to p within an accuracy of 0.2 with probability greater than 0.9

Answers

The value of k for which X will estimate to p within an accuracy of 0.2 with probability greater than 0.9 is {0.2^2 p(1-p)}/{1.645^2}

Given a random variable X where X ~ N(p, p(1-p)/k) and 0<=p<=1, we need to find the value of k for which X will estimate to p within an accuracy of 0.2 with probability greater than 0.9.

In general, if X ~ N(μ,σ²), then

P[|X-μ| < a] = 2Φ(a/σ) - 1

where Φ(z) is the standard normal cumulative distribution function.

Therefore, we can say that

P[|X-p| < 0.2] = 2Φ(0.2/√(p(1-p)/k)) - 1 ≥ 0.9

or 2Φ(0.2/√(p(1-p)/k)) ≥ 1.9

or Φ(0.2/√(p(1-p)/k)) ≥ 0.95

or 0.2/√(p(1-p)/k) ≥ Φ^(-1)(0.95)

where Φ^(-1)(z) is the inverse of the standard normal cumulative distribution function.

Therefore,  Φ^(-1)(0.95) = 1.6450.2/√(p(1-p)/k) ≥ 1.645

or k ≤ 0.2²p(1-p)/1.645²

From the above inequality, we get the maximum value of k for which X will estimate to p within an accuracy of 0.2 with probability greater than 0.9 is given by the formula:

k ≤{0.2^2 p(1-p)}/{1.645^2}

Therefore, the value of k for which X will estimate to p within an accuracy of 0.2 with probability greater than 0.9 is {0.2^2 p(1-p)}/{1.645^2}

Learn more about probability here:

https://brainly.com/question/31828911

#SPJ11

Find the area of the region that lies inside both the curves.
r = sin 2θ , r = sin θ

Answers

The area of the region that lies inside both the curves r = sin 2θ and r = sin θ is π/3 + (1/16)√3.

To find the area of the region that lies inside both the curves, we need to determine the limits of integration for the angle θ.

The curves r = sin 2θ and r = sin θ intersect at certain values of θ. To find these points of intersection, we can set the two equations equal to each other and solve for θ:

sin 2θ = sin θ

Using the trigonometric identity sin 2θ = 2sin θ cos θ, we can rewrite the equation as:

2sin θ cos θ = sin θ

Dividing both sides by sin θ (assuming sin θ ≠ 0), we have:

2cos θ = 1

cos θ = 1/2

θ = π/3, 5π/3

Now we have the limits of integration for θ, which are π/3 and 5π/3.

The formula for calculating the area in polar coordinates is given by:

A = (1/2) ∫[θ₁,θ₂] (r(θ))² dθ

In this case, the function r(θ) is given by r = sin 2θ. Therefore, the area is:

A = (1/2) ∫[π/3,5π/3] (sin 2θ)² dθ

To evaluate this integral, we can simplify the expression (sin 2θ)²:

(sin 2θ)² = sin² 2θ = (1/2)(1 - cos 4θ)

Now, the area formula becomes:

A = (1/2) ∫[π/3,5π/3] (1/2)(1 - cos 4θ) dθ

We can integrate term by term:

A = (1/4) ∫[π/3,5π/3] (1 - cos 4θ) dθ

Integrating, we get:

A = (1/4) [θ - (1/4)sin 4θ] |[π/3,5π/3]

Evaluating the integral limits:

A = (1/4) [(5π/3 - (1/4)sin (20π/3)) - (π/3 - (1/4)sin (4π/3))]

Simplifying the trigonometric terms:

A = (1/4) [(5π/3 + (1/4)sin (2π/3)) - (π/3 + (1/4)sin (4π/3))]

Finally, simplifying further:

A = (1/4) [(5π/3 + (1/4)√3) - (π/3 - (1/4)√3)]

A = (1/4) [(4π/3 + (1/4)√3)]

A = π/3 + (1/16)√3

Therefore, the area of the region that lies inside both the curves r = sin 2θ and r = sin θ is π/3 + (1/16)√3.

Learn more about curves here

https://brainly.com/question/30452445

#SPJ11

I WILL GIVE BRAINLIEST!!!!!!
Write in complete sentences to explain what a budget is, how to make one, and how to balance it.

Answers

A budget is the amount of money you have to spend on something, you make a budget by taking your amount of money and choosing how much you want to save and how much you want to spend on something, and you balance your budget by possibly splitting your money then using half of it as the budget and half of it as saving

1 Which is an arithmetic sequence?
F)2, 5, 9, 14, ...
G)100, 50, 12.5, 1.6, ...
H)3, 10, 17, 24,...
j) -2,-1,-1/2,-1/4

Answers

Answer:

H) 3, 10, 17, 24, ...

Step-by-step explanation:

An arithmetic sequence is when the difference of the terms is same

F)2, 5, 9, 14, ...

14 - 9 = 5, 9 - 5 = 4. 5-2 = 3

5 ≠ 4 ≠ 3, no

G)100, 50, 12.5, 1.6, ...

1.6 - 12.5 = -10.912.5 - 50 = -37.550 - 100 = -50

-10.9 ≠ -37.5 ≠ -50, no

H)3, 10, 17, 24,...

24-17 = 717 - 10 = 710 - 3 = 7

7 is the common difference, yes

j) -2,-1,-1/2,-1/4

-1/4 - (-1/2) = 1/4-1/2 - (-1) = 1/2-1 - (-2) = 1

1/4 ≠ 1/2 ≠ 1, no

pls help i’ll give brainliest

Answers

Answer:

The answer is 56.33 in decimal form but in fraction form the answer is 5633/

100

Step-by-step explanation:

6.55 x 8.6

How many students are enrolled in a course either in calculus, discrete mathematics, data structures, 7. or programming languages at a school if there are 507, 292, 312, and 344 students in these courses, respectively; 14 in both calculus and data structures; 213 in both calculus and programming languages; 211 in both discrete mathematics 558 and data structures; 43 in both discrete mathematics and programming languages; and no student may take calculus and discrete mathematics, or data structures and programming languages, concurrently

Answers

Answer:

974

Step-by-step explanation:

Let assume that:

The set of student that took part in Calculus be = C

Those that took part in discrete mathematics be = D

Let those that took part in data structures be = DS; &

Those that took part in Programming language be = P

Thus;

{C} = 507

{D} = 292

{DS} = 312

{P} = 344

For intersections:

{C ∩ DS} = 14

{C ∩ P} = 213

{D ∩ DS} = 211

{D ∩ P}  =43

{C ∩ D} = 0

{DS ∩ P} = 0

{C ∩ D ∩ DS ∩ P} = 0

According to principle of inclusion-exclusion;

{C ∪ D ∪ DS ∪ P} = {C} + {D} + {DS} + {P} - {C ∩ D} - {C ∩ DS} - {C ∩ P} - {D ∩ DS} - {D ∩ P} - {DS ∩ P}

{C ∪ D ∪ DS ∪ P} = 507 + 292 + 312 + 344 - 14 - 213 - 211 - 43 - 0

{C ∪ D ∪ DS ∪ P} = 974

Hence, the no of students that took part in either course = 974

Other Questions
Help this is clever 8.4 I need this by 7 A firm has a general-purpose machine, which has a book value of $300,000 and is worth $500,000 in the market. If the tax rate is 35 percent, what is the opportunity cost of using the machine in a project?A. $500,000B. $430,000C. $300,000D. $200,000 rogers pioneered community for learning that was an interesting large-group phenomena in the 1970s; unique about this experience was that group members _____ Suppose Sally wins the Publishers Clearinghouse $10 million sweepstakes (not the future value). The money is paid in equal annual installments of $500,000 over 20 years. If the appropriate discount rate is 5%, how much is the sweepstakes actually worth today? And if Publishers Clearinghouse offered you $5 million today, is that offer more or less valuable than the value of the annuity? A) $6,546,789; offer is less B) $10,000,000; offer is less C) $6,231,105; offer is less D) $4,121,212; offer is more E) none of the above pls pls pls :,) help me plssssssss no links no links NO LINKS pls and if you dont know dont answer because I really need help Combine these sentences to form a simple sentence with a compound subject.Our dog keeps barking. Our neighbor's dog keeps barking. What is ad targeting? How does it work? 4-5 sentences Q2) If the impulse response [] of an FIR filter is[] = [ 1] 2[ 4]a) (10 pt) Write the difference equation for the FIR filter.b) (10 pt) Obtain the impulse response of the system by using difference equation which you get in the previous step in MATLAB. Plot impulse response of the system, [] and compare with the result in (a). A plot of the Maxwell distribution of veloc- ities for a number of different gases measured at the same temperature shows that O, as the molecular mass increases, the spread of speeds widens. ,as molecular mass increases, fewer molecules have speeds close to their average speed. as the molecular mass increases, a higher pro- portion of molecules have very high speeds. as molecular mass increases, the distribution stays the same. as the molecular mass increases, the average speed decreases. Which letter labels the Tyrrhenian Sea? Can someone write a detailed gothic,scary setting description without using the word I Describe the changes in Anne's character as she grows up. (Anne of green gables)Plz answer fast...! Add f(x)=2x^3 and g(x) = log(x+4) + 100 Making Towers Bird-FriendlyThe lights on tall communication towers warn pilots to avoid the towers when flying at night. Unfortunately, the steady red lights often used on towers have the opposite effect on birds: the bright beams attract and confuse them. Instead of avoiding the lights, the birds fly directly toward them, crashing into the towers or becoming entangled in power lines. Millions of birds have died this way. However, there is a solution. Unlike steady lights, flashing or blinking lights don't attract birds. In response to demands from conservation groups, the Federal Aviation Administration in 2016 began requiring communication towers in the U.S. to use blinking lights instead of steady ones.What is the main idea of the passage?To save birds, communication towers have begun using blinking lights.Birds are attracted to the steady red lights on communication towers. Given the joint density f(x, y) = 1 for -y < x < y and 0 < y < 1, or 0 elsewhere, show that the random variables X and Y are uncorrelated but not independent. PLEASE HELP WILL MARK BRAINLIEST!!! No links and serious answers only!! 2. Write the equation of the line of best fit using the slope-intercept formula $y = mx + b$. Show all your work, including the points used to determine the slope and how the equation was determined. 3. What does the slope of the line represent within the context of your graph? What does the y-intercept represent? 4. Test the residuals of two other points to determine how well the line of best fit models the data. 5. Use the line of best fit to help you to describe the data correlation. 6. Using the line of best fit that you found in Part Three, Question 2, approximate how tall is a person whose arm span is 66 inches? 7. According to your line of best fit, what is the arm span of a 74-inch-tall person? 1. GASOLINE The table gives the cost of a gallon of gasoline at two stations. How much more does gasoline cost at Gas For Less than at Cut-Rate? Cut-Rate 2.x + 3.5 Gas for less V12 What is the midpoint of line segment RS with endpoints R(5, -10) and S(3, 6)? Which factor contributes the most to the rates of diffusion and effusion between two gases in a mixture?a)The size of the particles.b)The molar mass of the particles.c)The interactions between the particles.d)The relative volume of the particles to each other. I BEG FOR MY LIFE EZ PLS{{{{{{{PICTURE PROVIDED}}}}}}}}}